Question 4 (2 marks)
Justin works 14 hours at a normal pay rate of $24.80 per hour and 5 hours of overtime at
time and a half. How much should he be paid?
I
809 words
LE
English (Australia)

Answers

Answer 1

Answer:

554.7

Step-by-step explanation:

The pay=25.8*14+(25.8)*5*1.5=554.7


Related Questions

Given the following matrices, what 3 elements make up the first column of the product matrix DA?

Answers

We have to figure out what the product of DA is,

[tex]\begin{bmatrix}-1&2&3\\8&-4&0\\6&7&1\\ \end{bmatrix}\begin{bmatrix}1\\3\\5\\ \end{bmatrix}=\begin{bmatrix}a\\b\\c\\ \end{bmatrix}[/tex]

We know that,

[tex]\begin{bmatrix}a&b&c\\d&e&f\\g&h&i\\\end{bmatrix}\begin{bmatrix}x\\y\\z\\\end{bmatrix}=\begin{bmatrix}ax+by+cz\\dx+ey+fz\\gx+hy+iz\\\end{bmatrix}[/tex]

So,

[tex]a=-1\cdot1+2\cdot3+3\cdot5=-1+6+15=20[/tex]

[tex]b=8\cdot1+(-4)\cdot3+0\cdot5=8-12=-4[/tex]

[tex]c=6\cdot1+7\cdot3+1\cdot5=6+21+5=32[/tex]

So the solution is,

[tex]a,b,c=\boxed{20,-4,32}[/tex]

Hope this helps :)

PLEASE HELP!!!
Evaluate each expression.
(252) =

Answers

Answer:

1/5

Step-by-step explanation:

find lub and glb of the following set E={0.2, 0.23, 0.234, 0.2343, 0.23434, 0.234343,.....}​

Answers

The lub is 0.23[tex]\mathbf{\overline{43}}[/tex], while the glb is 0.2

The given set is presented as follows;

E = {0.2, 0.23, 0.234, 0.2343, 0.23434, 0.234343,...}

The least upper bound, lub, of a set, E, is known as the supremum of the set which is the number B such that all x ∈ E are of the value x  ≤ B, while there all y ∈ E has a x ∈ E such that t < x

Therefore;

The supremum, lub of the given set is 0.23[tex]\overline{43}[/tex]

The greatest lower bound, glb, b, also known as the infimum, is defined as follows;

b is the greatest lower bound if for all xE then x ≥ b

Given that b < t, then where x ∈ E, there exist a x < t

The glb of the given set is 0.2

Learn more about lub, supremum, glb, infimum, here;

https://brainly.in/question/23591741

if a stone is dropped from a cliff that is 122.5m high then its height in meters after t seconds is h=122.5-4.9t^2. find its velocity after 2s

Answers

Answer:

Step-by-step explanation:

Let t = 2

h = 122.5 - 4.9·2² = 122.5-19.6 = 102.9

If ‘BOXES’ is OBXSE, then BOARD is

Answers

9514 1404 393

Answer:

  OBADR

Step-by-step explanation:

The first two letters are swapped, and the last two letters are swapped.

  BOARD . . . becomes

  OBADR

Five subtracted from seven times a number is 9. What is the number?

A) Translate the statement above into an equation that you can solve to answer this question. Do not solve it yet. Use
x
as your variable.

The equation is _____________


B) Solve your equation in part [A] for
Answer:
x=

Answers

Answer:

18

Step-by-step explanation:

7-5=2

2x9=18

7x2=14

14-5=9

9=7n-5

9=7n-5
9=7(2)-5
9=14-5
9=9

I could be wrong an misunderstood the entire thing, however I can reassure you 18 is not the answer as the question states “Five subtracted from seven times a number is 9”, therefore the final answer in part A) needs to equal 9.

The probability distribution of a random variable X is given. x 1 2 3 4 P(X = x) 0.4 0.1 0.3 0.2 Compute the mean, variance, and standard deviation of X. (Round your answers to two decimal places.) mean variance standard deviation

Answers

Mean:

[tex]E(X) = \displaystyle \sum_{x\in\{1,2,3,4\}}x\,P(X=x) = 1\times0.4 + 2\times0.1 + 3\times0.3 + 4\times0.2 = \boxed{2.3}[/tex]

Variance:

[tex]\displaystyle V(X) = E\left((X-E(X))^2\right) = E(X^2) - E(X)^2 \\\\ E(X^2) = \sum_{x\in\{1,2,3,4\}}x^2\,P(X=x) = 1^2\times0.4 + 2^2\times0.1 + 3^2\times0.3 + 4^2\times0.2 = 6.7 \\\\ \implies V(X) = 6.7 - 2.3^2 = \boxed{1.41}[/tex]

Standard deviation:

[tex]\sigma_X = \sqrt{V(X)} = \sqrt{1.41} \approx \boxed{1.19}[/tex]

–21:(–2 – 5) + ( –14) + 6.(8 – 4.3)​

Answers

Did you mean -21(-2-5)+(-14)+6(8-4.3) ?
If that, the answer is 155.2:)) im sorry if im wrong:((

1. Ewa has 20 balls of four colors: yellow, green, blue, and black. 17 of them are not green, 5 are black, and 12 are not yellow. How many blue balls does Ewa have? (Use Gaussian elimination method).

Answers

Answer:

In a bag of balls, 1/4th are green, 1/8th are blue, 1/12th are yellow and the remaining 26 are white. How many balls are blue?

There are 4 colours of balls - green, blue, yellow and white.

Add (1/4)+(1/8)+(1/12) = (6/24)+(3/24)+(2/24) = 11/24 so the balance or (24–11)/24 = 13/24 = 26 white. Hence the total number of balls are 2*24 = 48.

Of the 48 balls, green are (1/4)*48 = 12, blue are (1/8)*48 = 6, yellow are (1/12)*48 = 4 and the rest, white are 26.

Check: Total number of balls = 12+6+4+26 = 48

Answer: 6 balls are blue....

please help! thanks!
find y.

Answers

Answer:

y = 4

Step-by-step explanation:

The ratio of the lengths of the sides of a 30-60-90 triangle is

1 : √3 : 2

The sides in this triangle are in the order:

y : 4√3 : x

y/1 = 4√3/√3

y = 4

Find the length of FT

Answers

Step-by-step explanation:

Hey there!

From the given figure;

Angle FVT = 43°

VT = 53

Taking Angle FVT as reference angle we get;

Perpendicular (p) = FT = ?

Base (b) = VT = 53

Taking the of tan;

[tex] \tan( \alpha ) = \frac{p}{b} [/tex]

Keep all values and simplify it;

[tex] \tan(43) = \frac{ft}{53} [/tex]

0.932515*53 = FT

Therefore, FT= 49.423.

Hope it helps!

Answer:

A. 49.42

Step-by-step explanation:

tan 43 = FT ÷ VT

0.932515086 = FT ÷ 53

49.42 = FT

In a high school graduating class of 300, 200 students are going to college, 40 are planning to work full-time, and 80 are taking a gap year.

a. These are mutually exclusive events.
b. These are not mutually exclusive events.
c. You should add their individual probabilities.
d. None of the above are true.

Answers

These are mutually exclusive events. So it is b

Find the quotient of 90 over -10

Answers

90/-10

= 9/-1

= -9

So, -9 is the quotient.

Hi! The answer is -9 because 90/-10 = -9
I hope this helps you, Goodluck! :)

If the white rod is 1/3, what color is the whole??

Answers

Answer:

brown

Step-by-step explanation:

it might be brown because it compelled

If x = 1, y = 7, and z = 15, determine a number that when added to x, y, and z yields
consecutive terms of a geometric sequence. What are the first three terms in the
geometric sequence?

Answers

You're looking for a number w such that the numbers

{1 + w, 7 + w, 15 + w}

form a geometric sequence, which in turn means there is a constant r for which

7 + w = r (1 + w)

15 + w = r (7 + w)

Solving for r, we get

r = (7 + w) / (1 + w) = (15 + w) / (7 + w)

Solve this for w :

(7 + w)² = (15 + w) (1 + w)

49 + 14w + w ² = 15 + 16w + w ²

2w = 34

w = 17

Then the three terms in the sequence are

{18, 24, 32}

and indeed we have 24/18 = 4/3 and 32/24 = 4/3.

Set up and evaluate the integral that gives the volume of the solid formed by revolving the region bounded by y=x^8 and y = 256 in the first quadrant about the y-axis.

Answers

Using the shell method, the volume integral would be

[tex]\displaystyle 2\pi \int_0^2 x(256-x^8)\,\mathrm dx[/tex]

That is, each shell has a radius of x (the distance from a given x in the interval [0, 2] to the axis of revolution, x = 0) and a height equal to the difference between the boundary curves y = x ⁸ and y = 256. Each shell contributes an infinitesimal volume of 2π (radius) (height) (thickness), so the total volume of the overall solid would be obtained by integrating over [0, 2].

The volume itself would be

[tex]\displaystyle 2\pi \int_0^2 x(256-x^8)\,\mathrm dx = 2\pi \left(128x^2-\frac1{10}x^{10}\right)\bigg|_{x=0}^{x=2} = \boxed{\frac{4096\pi}5}[/tex]

Using the disk method, the integral for volume would be

[tex]\displaystyle \pi \int_0^{256} \left(\sqrt[8]{y}\right)^2\,\mathrm dy = \pi \int_0^{256} \sqrt[4]{y}\,\mathrm dy[/tex]

where each disk would have a radius of x = ⁸√y (which comes from solving y = x ⁸ for x) and an infinitesimal height, such that each disk contributes an infinitesimal volume of π (radius)² (height). You would end up with the same volume, 4096π/5.

The volume of the solid formed by revolving the region bounded by y=x^8 and y = 256 in the first quadrant about the y-axis is 4096π/5 cubic units.

What is integration?

It is defined as the mathematical calculation by which we can sum up all the smaller parts into a unit.

We have a function:

[tex]\rm y = x^8[/tex]   or

[tex]x = \sqrt[8]{y}[/tex]

And  y = 256

By using the vertical axis of rotation method to evaluate the volume of the solid formed by revolving the region bounded by the curves.

[tex]\rm V = \pi \int\limits^a_b {x^2} \, dy[/tex]

Here a = 256, b = 0, and [tex]x = \sqrt[8]{y}[/tex]

[tex]\rm V = \pi \int\limits^{256}_0 {(\sqrt[8]{y}^2) } \, dy[/tex]

After solving definite integration, we will get:

[tex]\rm V = \pi(\frac{4096}{5} )[/tex]  or

[tex]\rm V =\frac{4096}{5}\pi[/tex] cubic unit

Thus, the volume of the solid formed by revolving the region bounded by y=x^8 and y = 256 in the first quadrant about the y-axis is 4096π/5 cubic units.

Learn more about integration here:

brainly.com/question/18125359

Please helps fill in the charts
A and b
With order of pairs

Answers

Answer:

...

Step-by-step explanation:

seeee the above picture

work out missing angle following polygons​

Answers

Answer:

x = 150°

Step-by-step explanation:

Interior angle of a hexagon = 120° and interior angle of a square = 90°

so remaining angle, 360-120-90 = 150°

Find the output, hhh, when the input, ttt, is 353535.
h = 50 - \dfrac{t}{5}h=50−
5
t

h, equals, 50, minus, start fraction, t, divided by, 5, end fraction
h=

Answers

9514 1404 393

Answer:

  43

Step-by-step explanation:

Put the value where t is and do the arithmetic.

  h = 50 -t/5

  h = 50 -35/5 = 50 -7 = 43

The output, h, is 43 when the input is 35.

Answer:

43

Step-by-step explanation:

The answer is 43 on Khan :)

I need help completing this problem ASAP

Answers

Answer:

D. [tex]3x\sqrt{2x}[/tex]

Step-by-step explanation:

The problem gives on the following equation:

[tex]\sqrt{32x^3}+-\sqrt{16x^3}+4\sqrt{x^3}-2\sqrt{x^3}[/tex]

Alongside the information that ([tex]x\geq0[/tex]).

One must bear in mind that the operation ([tex]\sqrt[/tex]) indicates that one has to find the number that when multiplied by itself will yield the number underneath the radical. The easiest way to find such a number is to factor the term underneath the radical. Rewrite the terms under the radical as the product of prime numbers,

[tex]\sqrt{2*2*2*2*2*x*x*x}-\sqrt{2*2*2*2*x*x*x}+4\sqrt{x*x*x}-\sqrt{2*x*x*x}[/tex]

Now remove the duplicate factors from underneath the radical,

[tex]2*2*x\sqrt{2x}-2*2*x\sqrt{x}+4x\sqrt{x}-2x\sqrt{x}[/tex]

Simplify,

[tex]4x\sqrt{2x}-4x\sqrt{x}+4x\sqrt{x}-x\sqrt{2x}[/tex]

[tex]3x\sqrt{2x}[/tex]

Hello Pls help and thanks

Answers

Answer:

c.) in the correct answer

factorise m^2 - 12 m + 24

Answers

Answer:

(m-6+2root3)(m-6-2root3)

Step-by-step explanation:

m^2 - 12m +36 -12

= (m-6)^2 - 12

= (m-6+2root3)(m-6-2root3)[root 12 = 2root3]

Why wouldn't you use division to find an equivalent fraction for 7/15

Answers

Answer:

This depends whether you want to make the fraction bigger or smaller.

Step-by-step explanation:

If you want to the the fraction into something smaller than it already is, you would use division because when you divide something, you get a smaller number.

However, if you want to make the fraction bigger, then you would multiply.

Hope this helps! :)

Answer:

Because 7 is a prime number which means it can only divide by itself and one so you cannot divide seven but you can divide 15.

Step-by-step explanation:

△DOG ~△?
Complete the similarity statement and select the theorem that justifies your answer.
**If they are not similar, select "none" for both parts

Answers

9514 1404 393

Answer:

nonenone

Step-by-step explanation:

The reduced side ratios, shortest to longest are ...

  AC : AT : CT = 8 : 9 : 15

  OD : OG : DG = 5 : 6 : 10

These are different ratios, so the triangles are not similar.

Help me plz 20 points to who ever gets it right

Answers

Step-by-step explanation:

2., 3., 4., 5.

yes, you had the right idea to calculate the half distances between the coordinates. just create the absolute values of the full distance before cutting it in half.

you need to remember : we have to go this half distance from one point to the other (meaning adding our subtracting the half distance to/from the starting point).

2.

(-4, 6) to (10, -10)

in x the distance is 10 - -4 = 14. half is 7.

in y the distance is |-10 - 6| = |-16| = 16. half is 8.

so the midpoint is

(-4 + 7, 6 - 8) = (3, -2)

remember, to go the half distance in the direction towards the second point (so we have to choose properly, when to use "+" and "-" depending on the change of the coordinate : from -4 to 10 we have to add, from 6 to -10 we have to subtract, of course).

3.

(-3, -8) to (-6.5, -4.5)

in x distance : -3 - -6.5 = 3.5. half is 1.75

in y distance : -8 - -4.5 = |-3.5| = 3.5. half is 1.75

midpoint is

(-3 - 1.75, -8 + 1.75) = (-4.75, -6.25)

4.

(3, 7) to (-8, -10)

x : 3 - -8 = 11. half is 5.5

y : 7 - -10 = 17. half is 8.5

midpoint is

(3 - 5.5, 7 - 8.5) = (-2.5, -1.5)

5.

(-6, -13) to (-6.4, -3.8)

x : -6 - -6.4 = 0.4. half is 0.2

y : -13 - -3.8 = |-9.2| = 9.2. half is 4.6

midpoint is

(-6 - 0.2, -13 + 4.6) = (-6.2, -8.4)

6.

(-1, 7) to (5, 1)

x : -1 - 5 = |-6| = 6. 1/3 is 2.

y : 7 - 1 = 6. 1/3 is 2.

1/3 from C to D

(-1 + 2, 7 - 2) = (1, 5)

7.

2/3 of the way from D to C is the same point as in 6. (1/3 from C to D).

again

(1, 5)

8.

2/3 of the way from C to D.

so, we need to double what we added in 6.

(-1 + 4, 7 - 4) = (3, 3)

9.

1/3 of the way from D to C is the same point as in 8. (2/3 of the way from C to D).

again

(3, 3)

10.

exactly. Pythagoras.

the square root of the sum of the squares of the coordinate differences.

distance = sqrt((x1 - x2)² + (y1 - y2)²)

11.

(6, 8) to (-1, 8)

distance = sqrt((6 - -1)² + (8 - 8)²) = sqrt(49) = 7

12.

(5, -6) to (5, 6)

sqrt((5-5)² + (-6-6)²) = sqrt(144) = 12

13.

(-2, 0) to (11, 0)

sqrt((-2 - 11)² + (0-0)²) = sqrt(169) = 13

14.

(1, -5) to (9, 1)

sqrt((1-9)² + (-5 - 1)²) = sqrt(64 + 36) = sqrt(100) = 10

15.

ST and MT are basically the same equation.

MT is half of ST.

ST equation based on 2 points :

y – yS={(yT – yS)/(xT – xS)}(x – xS)

M = (xS + (xT - xS)/2, yS +(yT - yS)/2)

so, let's put that into the general equation :

y - yM={(yT - yM)/(xT - xM)}(x - xM)

y - (yS +(yT - yS)/2) = {(yT - (yS +(yT - yS)/2))/(xT - (xS + (xT - xS)/2))}(x - (xS + (xT - xS)/2))

16.

the two corners farthest away are (5, 10) and (9, 6).

what distance from (0, 0) is now bigger ?

since it is (0, 0), we can skip the 0s and just sum up the squares of the coordinates.

5² + 10² = 125

9² + 6² = 117

so, the corner (5, 10) is the farthest away.

the ratio of sadia's age to her father's age is 3:6. The sum of their age is 96 .What is sadia's age​

Answers

We have,

[tex]a:b=3:6,a+b=96[/tex]

Introduce variable [tex]x[/tex] such that [tex]a=3x,b=6x[/tex]

The sum [tex]a+b=96[/tex] is therefore [tex]9x=96\implies x=10.\overline{6}[/tex]

So,

[tex]a=3\cdot10.\overline{6}=\boxed{32}[/tex] (sadia's age)

[tex]b=6\cdot10.\overline{6}=\boxed{64}[/tex] (father's age)

Hope this helps :)

A lawyer commutes daily from his suburban home to his midtown office. The average time for a one-way trip is 24 minutes, with a standard deviation of 3.8 minutes. Assume the distribution of trip times to be normally distributed.
(a) What is the probability that a trip will take at least ½ hour?
(b) If the office opens at 9:00 A.M. and he leaves his house at 8:45 A.M. daily, what percentage of the time is he late for work?
(c) If he leaves the house at 8:35 A.M. and coffee is served at the office from 8:50 A.M. until 9:00 A.M., what is the probability that he misses coffee?
​(d) Find the length of time above which we find the slowest 10​% of trips.
(e) Find the probability that 2 of the next 3 trips will take at least one half
1/2 hour.

Answers

Answer:

Step-by-step explanation:

a) Probability-Above 30 min = 5.72% = .0572

b) Probability-Above 15 min =  99.11% = .9911

c) *Probability-Between  1 - 59.49% = .4051

d) 19.136 minutes  z = -1.28

a) The probability that trip will take at least 1/2 hour will be 0.0606.

b) The percentage of time the lawyer is late for work will be 99.18%.

c) The probability that lawyer misses coffee will be 0.3659.

d) The length of time above which we find the slowest 10​% of trips will be 0.5438.

e) The probability that exactly 2 out of 3 trips will take at least one half

1/2 hour is 0.0103.

What do you mean by normal distribution ?

A probability distribution known as a "normal distribution" shows that data are more likely to occur when they are close to the mean than when they are far from the mean.

Let assume the time taken for a one way trip be x .

x ⇒ N( μ , σ ²)

x  ⇒ N( 24 , 3.8 ²)

a)

The probability that trip will take at least 1/2 hour or 30 minutes will be :

P ( x ≥ 30)

= P [ (x - μ) / σ ≥ (30 - μ) / σ ]

We know that , (x - μ) / σ = z.

= P [ z ≥ (30 - 24) / 3.8)]

= P [ z ≥ 1.578 ]

= 1 - P [  z ≤ 1.578 ]

Now , using the standard normal table :

P ( x ≥ 30)

= 1 - 0.9394

= 0.0606

b)

The percentage of the time the lawyer is late for work will be :

P ( x  ≥ 15)

= P [ z ≥ -2.368 ]

= P [  z ≤ 2.368]

= 0.9918

or

99.18%

c)

The probability that lawyer misses coffee :

P ( 15 < x < 25 ) = P ( x < 25 ) - P ( x < 15)

= P [  z < 0.263] - P ( z < -2.368)

or

= 0.3659

d)

The length of time above which we find the slowest 10​% of trips :

P( x ≥ X ) ≤ 0.10

=  0.5438

e)

Let's assume that y represents the number of trips that takes at least half hour.

y ⇒ B ( n , p)

y ⇒ B ( 3 , 0.0606)

So , the probability that exactly 2 out of 3 trips will take at least one half

1/2 hour is :

P ( Y = 2 )

= 3C2 × (0.0606)² × ( 1 - 0.0606)

= 0.0103

Therefore , the answers are :

a) The probability that trip will take at least 1/2 hour will be 0.0606.

b) The percentage of time the lawyer is late for work will be 99.18%.

c) The probability that lawyer misses coffee will be 0.3659.

d) The length of time above which we find the slowest 10​% of trips will be 0.5438.

e) The probability that exactly 2 out of 3 trips will take at least one half

1/2 hour is 0.0103.

Learn more about normal distribution here :

https://brainly.com/question/26822684

#SPJ2

The answer pl shhaoksngausinxbbs pls

Answers

Answer:

D. 3

Step-by-step explanation:

A triangle can be defined as a two-dimensional shape that comprises three (3) sides, three (3) vertices and three (3) angles.

Simply stated, any polygon with three (3) lengths of sides is a triangle.

In Geometry, a triangle is considered to be the most important shape.

Generally, there are three (3) main types of triangle based on the length of their sides and these include;

I. Equilateral triangle: it has all of its three (3) sides and interior angles equal.

II. Isosceles triangle: it has two (2) of its sides equal in length and two (2) equal angles.

III. Scalene triangle: it has all of its three (3) sides and interior angles different in length and size respectively.

In Geometry, an acute angle can be defined as any angle that has its size less than ninety (90) degrees.

Hence, we can deduce that the greatest number of acute angles that a triangle can contain is three (3) because the sum of all the interior angles of a triangle is 180 degrees.

Rotation 90° counterclockwise around the origin of the point (-8,1)​

Answers

(-1, -8). When we rotate to the fourth quadrant, we make both of the coordinate values negative.

The HCF of two numbers is 175. The LCM of these two numbers is 12600. Both numbers are greater than their HCF. Find the two numbers

Answers

Answer:

Hello,

Answer : 1400 and 1575

Step-by-step explanation:

Let's say a and b the ywo numbers

[tex]HCF(a,b)=a\vee b=175=5^2*7\\LCM(a,b)=a\wedge b=12600\\\\a*b=(a\vee b)*(a\wedge b)=(2^3*3^2*5^2*7)*(5^2*7)=2^3*3^2*(5^2*7^2)^2\\\\Both\ numbers\ are\ greater\ than\ their HCF\\a=175*k_1\\b=175*k_2\\\\k_1=2^3\ and\ k_2=3^2\\\\a=175*2^3=1400\\b=175*3^2=1575\\\\[/tex]

Other Questions
Simplificar expresiones algebraicas The equation of line u is y = 9/2x+1. Line v is perpendicular to u. What is the slope of line v Find the equation of the line through the points (-5,11) and (2,-3)y= A recipe calls for 2 1/2 tablespoons of oil and 3/4 tablespoons of vinegar. What is the ratio of oil to vinegar in this recipe? Two line segments are parallel, and each is 8 centimeters long. How many rectangles can be constructed using this information?A.0B.11OC. 2OD4OEInfinitely many Bert has blonde hair (b). Molly has brown hair (B), but her genotype is Bb, meaning she carries a recessive blonde allele. What is the likelihood of Bert and Molly's second child having brown hair? What happens if the products of digestion of proteins in a boiled egg are more than what the body needs ? Biomagnification results in higher amounts of toxic substances for organisms on the food chain True False I NEED HELP ASAP ON THIS ITS VERY IMPORTANTWhich one of these documents told Europe to stop trying to create new colonies in the Western Hemisphere?A. Adams-Ons TreatyB. Border CompromiseC. Manifest DestinyD. Monroe Doctrine Bob's truck averages 23 miles per gallon. If Bob is driving to his mother's house, 72 miles away, how many gallons of gas are needed? Round to the nearest tenth. Which sentence uses a comma correctly to set off an introductory phrase? why do foreign people visit national heritage of nepal? In 4 points. In 500 words decribe fluently the imagery in the peom Grey day at waterval farm by lionel murcott Which of the following is in standard form of quadratic? f(x)=-5x^(2)-3x+9 f(x)=-5(x+4)(x-2) f(x)=-5(x-1)^(2)+6 f(x)=2x-8 what is soical evil define it Which statement best explains the two possibleinterpretations of the sonnet? which of these is a cash crop A. cucumbersb. grassc. latexd. strawberries Granfield Company is considering eliminating its backpack division, which reported an operating loss for the recent year of $42,200. The division sales for the year were $963,800 and the variable costs were $477,000. The fixed costs of the division were $529,000. If the backpack division is dropped, 40% of the fixed costs allocated to that division could be eliminated. The impact on Granfield's operating income for eliminating this business segment would be:__________.a. $275,200 increaseb. $486,800 decreasec. $211,600 increased. $275,200 decreasee. $486,800 increase At what angle torque is half of the max An isosceles right triangle has a hypotenuse that measures 42 cm. What is the area of the triangle?PLEASE HELP